After monitoring blood levels of lycopene (a nutrient found in some fruits and vegetables) in 1,000 middle-aged study...

jingjingxiao11111@gmail.com on June 4, 2020

Why is A correct?

Hi I didn't pick A because I thought not necessarily people get their lycopene from fruits and vegetables so fruits and vegetables rich in lycopene more likely to contain more nutrients for reducing stroke would not weaken the argument. Did I overthink this question? Thanks. Please explain A as the correct answer.

Reply
Create a free account to read and take part in forum discussions.

Already have an account? log in

Skylar on June 17, 2020

@jingjingxiao11111@gmail.com, happy to help!

The passage describes a study in which lower levels of lycopene are found to be correlated with a higher risk of stroke. The passage concludes that this must mean lycopene causes a reduction in stroke risk. In other words, it identifies lycopene as the cause and reduction in stroke risk as the effect. We are asked to weaken this argument.

(A) incorporates information introduced in the parentheses of the passage- that lycopene is found in some fruits and vegetables- to provide an alternative explanation for the reduction in stroke risk. By providing this alternate cause for the effect, (A) weakens the argument's claim that lycopene is the cause. You point out that we don't necessarily know the people in the study are getting their lycopene from fruits/vegetables, but this does not eliminate (A) as the best answer. Remember, we are looking for the answer choice that could most weaken the argument. It is very possible (and even insinuated in the passage) that the people could be obtaining their lycopene from fruits/vegetables, especially since the study is looking at 1,000 people over 12-years and the only mention of where lycopene is found is in fruits/veggies. Therefore, (A) is correct.

(B) is incorrect because it is consistent with the passage's logic.
(C) is incorrect because we are only given information on/concerned with middle-aged people. Remember, we should not incorporate our real world knowledge about who has more strokes, as the passage does not mention the risk of stroke for young adults.
(D) is incorrect because it is consistent with the passage and does not add anything.
(E) is incorrect because it is unspecific and irrelevant.

Does that make sense? Hope it helps! Please let us know if you have any other questions!